LSAT and Law School Admissions Forum

Get expert LSAT preparation and law school admissions advice from PowerScore Test Preparation.

 Administrator
PowerScore Staff
  • PowerScore Staff
  • Posts: 8917
  • Joined: Feb 02, 2011
|
#81499
Complete Question Explanation

Weaken. The correct answer choice is (D).

Answer choice (A):

Answer choice (B):

Answer choice (C):

Answer choice (D): This is the correct answer choice.

Answer choice (E):

This explanation is still in progress. Please post any questions below!
 LustingFor!L
  • Posts: 80
  • Joined: Aug 27, 2016
|
#34291
Please confirm my reasoning is correct. I chose "D", because the conclusion of "some scientists" was that its unlikely the melting process has begun since water temperatures have not changed in a century. D weakens by showing that melting process can occur without a temperature change.
 Luke Haqq
PowerScore Staff
  • PowerScore Staff
  • Posts: 742
  • Joined: Apr 26, 2012
|
#34361
Hi!

Yes, your reasoning is correct. Answer (D) notes that the temperature can remain constant even though the warming process is occurring.

Second, you're right to look to the "some scientists" part, which the question stem asks you to weaken. More broadly, you could think of this as a causal question, and PowerScore has a good discussion of how to attack these. If you have the full-length books, see 3-7, "How to Attach a Causal Conclusion." What answer (D) does is it weakens the some scientist's causal argument by showing that the cause (melting) can exist without the effect (rising water temperatures).
 bk1111
  • Posts: 103
  • Joined: Apr 22, 2017
|
#37341
Hello, I got this answer right but I am having a hard time eliminating C. Can someone explain how air temperature relates to the stimulus and how I can confidently eliminate this answer choice?
 nicholaspavic
PowerScore Staff
  • PowerScore Staff
  • Posts: 271
  • Joined: Jun 12, 2017
|
#37685
Hi bk,

Remember that in Weakening questions, there can often be irrelevant information in wrong answer choices. The short anwer is that air temps do not relate at all to the water temperature discussion. These are also refered to as "Out-of-Scope" answer traps in our materials. Whenever you see new information like this introduced into a Weakening answer choice, feel confident that it is a classical incorrect answer trap and mark it as a loser.

Thanks for the great question! :-D
 ynam5401
  • Posts: 3
  • Joined: Nov 25, 2019
|
#77657
Can you please explain why answer choice B is incorrect.
I chose B, because it introduced new information 'global sea levels' as an indication of the melting process. Therefore this answer choice weakens the scientists' contention on unwavering water temperature.
 Frank Peter
PowerScore Staff
  • PowerScore Staff
  • Posts: 99
  • Joined: May 14, 2020
|
#77719
Hi ynam,

While it is true that we're allowed to bring in new information to attack an argument in the weakener context, remember that we need to focus on the specific claim that is being made and the evidence that is used to support it. Here, the claim is that it is unlikely that the melting process has begun, because temperatures in the polar seas are the same as a century ago.

While it might be true that the overall effect of the melting process will be an increase in global sea levels, the issue here is over whether the melting process has begun or has not begum. (B) isn't really addressing this issue - it's more along the lines of what the net effect might be once the melting process has already happened.

(D) gives a reason to think that the scientists may not be correct - that the temperature of the water remaining the same as it was a century ago may not necessarily be conclusive evidence that the melting process has begun or not begun.

Get the most out of your LSAT Prep Plus subscription.

Analyze and track your performance with our Testing and Analytics Package.